Consider the equation 5 +x=n.
What must be true about any value of x if n is a negative number?
Explain your answer.
Provide an example that would satisfy this condition.
Explain how you know your equation is correct.

Answers

Answer 1

Answer:

It's C.

Step-by-step explanation:

Any value of x must have a bigger negative number than 5. In this case, formally you say that all x values must be LESS than 5 only.


Related Questions

Help!!!!!!!!!!!!!!!!!!!!!!!

Answers

the area would be 204 cm

Based on the definition of independence, which THREE statements are correct?
P(A) ,
A)
1
1
and B)
12
Events A and B are independent.
12
35
Events A and B are independent.
P(A) = 4, P(B) = , P(A and B) =
B)
1
1
P(A)
A B
10
Events A and B are independent.
P(A) = , P(B) = , P(A and B) =
7.
12
D)
Events A and B are dependent.
PLA) = }, PB) - to, P(A and B) - 1 2 3
E)
Events A and B are independent.

Answers

Answer: A, D, and E

Step-by-step explanation:

Sybil has 1/2 of pizza left over. She wants to share the pizza with 3 of her friends. What fraction of the original pizza does sybil and her friends each receive.

Answers

Answer:

1/6

Step-by-step explanation:

Double the original amount: 1/3. Which is 1/6, not 2/6, if it was 2/6 then people waould get 1/3 instead (not equal).

how much would be in your savings account in 11 years after depositing $150 today if the bank pays 7% a year

Answers

The answer is 315.73

simplify 1/a+b + 1/a-b + 2b/a^2-b^2​

Answers

Answer:

−a^2 b^2 + 2a + 2b

_________________

             a^2

Step-by-step explanation:

Please mark as brainliest if answer is right  

Have a great day, be safe and healthy  

Thank u  

XD  

Johanna can jog 24 feet in 5 seconds. If she jogs at the same rate, how many feet can she jog in 8 seconds? PLSS help!

Answers

38.4? Since if she runs 24 feet in five seconds add that two times she would run 48 feet in ten second so.. divide 24 with 5 and get 4.8. So 48 minus 9.6 makes 38.4.

express 0.6 + 0.47 in the form p/q where p and q are integers q are not equal to 0​

Answers

Answer:

[tex]0.6 + \bar 0.47 = \frac{532}{495}[/tex]

Step-by-step explanation:

Given

[tex]0.6 + \bar 0.47[/tex]

Required

Express as x/y

Let

[tex]x = \bar 0.47[/tex]

This implies that:

[tex]x = 0.4747[/tex] --- (1)

Multiply by 100

[tex]100x = 47.4747[/tex] --- (2)

Subtract 1 from 2

[tex]100x - x = 99x[/tex]

This gives

[tex]47.4747 - 0.4747 = 47[/tex]

So:

[tex]99x = 47[/tex]

Solve for x

[tex]x = \frac{47}{99}[/tex]

This implies that:

[tex]\bar 0.47 = \frac{47}{99}[/tex]

[tex]0.6 + \bar 0.47[/tex] becomes

[tex]0.6 + \bar 0.47 = 0.6 + \frac{47}{99}[/tex]

Express 0.6 as fraction

[tex]0.6 + \bar 0.47 = \frac{6}{10} + \frac{47}{99}[/tex]

Take LCM and solve

[tex]0.6 + \bar 0.47 = \frac{99*6+10*47}{990}[/tex]

[tex]0.6 + \bar 0.47 = \frac{1064}{990}[/tex]

Simplify

[tex]0.6 + \bar 0.47 = \frac{532}{495}[/tex]

Given : M, N, O are midpoints of AB, BC, AC Complete the following statements
MNOA is a

Answers

the answer is trapezoid

Which fraction is greater 2/5 or 4/10

Answers

They are equal. 2/5 is 4/10.

Someone please help me, this test determines if I pass or not.

Answers

Answer:

Your answer is C :)

arrange the following fraction in descending order 1/2 5/7 1/4 or 1/8 11/12​

Answers

Answer:

11/12, 5/7, 1,8, 1/4, 1/2

Step-by-step explanation:


Write -9 9/16 as a decimal

Answers

Answer:

-8.4375

Step-by-step explanation:

convert -9 9/16 to a improper fraction.

-9 x 16 = -144  

-144+9= -135

-135 divided by 16 = -8.4375

Can somebody please help me out please my grads are bad

Answers

The answer would be 729

3x-8= 4
What is the first and second steps in solving this equation?

a. Multiply each side by 3

b. Divide each side by 3
c.Add 8 to each side

d. Subtract 8 from each side

Answers

I believe the answer is D

!!!!Plz Help!!!! How does the slop of AB compare with the slope of the parallel line as you move points A, B, and C? What conclusion can you draw about the relationship between the slopes of parallel lines?​

Answers

Answer:

As points, A and B move, the slope of the two lines change but remain equal to each other. As point C moves, the slope does not change but remains equal to each other since the line through C is drawn parallel to AB.

I can canclude that slope of any two parallel lines are equal

Step-by-step explanation:

Evaluate each limit given that limit x->2 f(x)=9.

limit x->2 1/3[f(x)]^2= 27✔️
limit x->2 3f(x)+5= 32✔️
limit x->2 [f(x)]3/2= 27✔️

Answers

Answer:

27

32

27

Step-by-step explanation:

1) limit x->2 1/3[f(x)]^2= 27

2) limit x->2 3f(x)+5= 32

3) limit x->2 [f(x)]3/2= 27/2

What is limit?

"A limit is the value that a function approaches as the input approaches some value."

Limit properties:

If limit f(x) = m and constant 'a',

limit a (f(x)) = a × m

limit (f(x))^n = (m)^n

limit (a) = a

limit (f(x) + a) = m + a

Given: limit x->2 f(x)=9.

We need to evaluate each limit.

1) limit x->2 1/3[f(x)]^2

⇒ [tex]\lim_{x \to 2} \frac{1}{3} (f(x))^2[/tex]

= [tex]\frac{1}{3} \lim_{x \to 2} (f(x))^2[/tex]

= [tex]\frac{1}{3} (\lim_{x \to 2} f(x))^2[/tex]

= [tex]\frac{1}{3} \times (9)^2[/tex]

= [tex]\frac{81}{3}[/tex]

= 27

2) limit x->2 3f(x)+5

⇒ [tex]\lim_{x \to 2} (3f(x)+5)[/tex]

= [tex]\lim_{x \to 2} 3f(x)+ \lim_{x \to 2} 5[/tex]

= [tex]3\lim_{x \to 2} f(x) + 5[/tex]

= [tex](3 \times 9)+5[/tex]

=[tex]\bold{32}[/tex]

3) limit x->2 [f(x)]3/2

⇒ [tex]\lim_{x \to 2} (f(x))\frac{3}{2}[/tex]

= [tex]\frac{3}{2} \times (\lim_{x \to 2} f(x))[/tex]

= [tex]\frac{3}{2} \times 9[/tex]

= 27/2

Learn more about limit here:

https://brainly.com/question/1619243

#SPJ2

There were 8 pizzas at the pizza party for two families. The Houston family ate 2 2/3 pizzas.
The Ozark family ate 2 5/6 pizzas.
What whole number is the closest estimate of how many pizza was left

Answers

Answer:2 pizzas

Step-by-step explanation:

2 2/3 and 2 5/6 can both be rounded to 3. Double that and you get 6. Since there were 8 pizzas originally,subtract 6 from 8 to get 2 remaining pizzas.

Answer:

2 pizzas

Step-by-step explanation:

please please help me!!!!!!!​

Answers

Answer: there is your answer and how

Step-by-step explanation:

Help pls I rlly need help it’s important

Answers

Answer:

2. 112

3. 78

5. 672

7.  149.8

9. 17.5

10. 19360

11. 64 000

13. 29

14. 350240

15. 386

16. 224

17. 0.9071847

Step-by-step explanation:

PLEASE HELP I DON’T UNDERSTAND

Answers

Answer:

yea I don't understand either kid, it's upside down.

Step-by-step explanation:

Solve the system by substitution.
x = - 4y - 5
2x + 10y = 8

Answers

Answer:

x = -41

y = 9

Step-by-step explanation:

Begin by multiplying the top equation by 2

2x = -8y - 10

The second equation requires a little more care.

Start by moving 10y to the right. Normally the y value is on the left, but we are going to be substituting for 2x.

2x = - 10y + 8

What I've done is not the usual practice, but there is nothing wrong with it.

Now since both right sides are equal to 2x, you can substitute for them

-8y - 10 = -10y + 8                Add 10y to both sides

-8y + 10y - 10 = 8

2y - 10 = 8                           Add 10 to both sides

2y = 8+10

2y = 18                                 Divide by 2

y = 18/2

y = 9

Use the second original equation to solve for x

2x + 10y = 8

2x + 10*9 = 8                    

2x + 90 = 8                         Subtract 20 from both sides

2x = 8 - 90

2x =  - 82                              Divide by 2

x = -822/2

x = - 41

If there were 30 questions on a test, and John got 24 of the correct, what is the percentage of the number of questions John got correct?

Answers

Answer: he got 80/100 or 80% right

Step-by-step explanation:

24/30 = 12/15

12/15 as a percent:

100/15 = 6.66666666666667

12 x 6.66666666667

15 x 6.66666666667

= 80/100 or 80%

write an addition equation for a triangle with side lengths of 3, √2, √7

Answers

Answer:

3² = (√2)² + (√7)²

Step-by-step explanation:

The given sides to us are 3 , √2 and √7. We can check if these are the sides of right angled triangle.

For that , we can use Pythagoras Theorem , which is = + . Here 3 is the longest side so , it can be hypotenuse.

=> h² = p² + b²

=> 3² = (√2)² + (√7)²

=> 9 = 2 + 7

=> 9 = 9

Hence the ∆ is right angled .

Hence the addition equation is 3² = (√2)² + (√7)² .

Present
earch and writ
storie​

Answers

Answer:

What did you say?? i dont know what your trying to say?

Answer:

?

Step-by-step explanation:

18
The measures tanker are shown
12 inches
10 inches
8 inches
Enter the come of the coolerin cubie inches

Answers

Answer:

78

Step-by-step explanation:

Explain how to graph the following including asymptotes and key features. f(x)=3^x+2

Answers

Answer:

y=2

Step-by-step explanation:

Help me plz what is the theme


Making new friends is too difficult, so you shouldn’t try.


You have to take risks in order to get rewards.


Jared felt lonely and sad because he was too afraid to take a chance.


Honesty is always the best policy.

Answers

Do you have the text like as in the passage ? If so upload it

Answer:

There is no passage/short story etc. to base this question off of, but if you copy and paste it I'll come back and edit my answer :)

can anybody help me please

Answers

Answer:

Is B

Step-by-step explanation:

Answer:

      B

Step-by-step explanation:

calculate the size of angle x​

Answers

Answer:

16

Step-by-step explanation:

Somebody know the answer?

Answers

Your teacher does know the answer
Other Questions
Kari is a limited partner in Lizard Partnership. This year, Kari's share of partnership ordinary income is $20,000, and she received a cash distribution of $30,000. Kari's tax basis in her partnership interest at the beginning of the year was $50,000. Her marginal tax rate is 22 percent. Kari qualifies for the QBI deduction, without regard to the wage or taxable income limitations.a. Calculate the tax cost of Kari's partnership earnings this year Tax cost b. Compute Kari's after-tax cash flow from her partnership activity this year After-tax cash flow c. Compute Kari's tax basis in her partnership interest at the ending of the year. Assume no change in her share of partnership during the year. If the regular price of an item is x dollars, what is the discount price indollars?7.bThe price of the is x dollars is $ 3+6+2=61 2+8+3=31 4+7+4=42 5+6+4=62 8+7+4=????? need help fast !!!!!!!!!!!!!!!!!!!!! Discuss the quotations that open and close the story. How do these quotes contribute to the meaning of the story? Cite evidence from the text in your response A number X rounded to 2 decimal places is 7.42. Write down the error interval for X a person that supported voting rights for women *delegatesantocivil right activistsuffragist Henry's paycheck was 3,200$. He spent 25% of his paycheck on rent and 0.37 of his paycheck on other expenses. How much money did henry have left? Can public art instill a sense of civic identity? Brainliest if correct LINKS WILL BE REPORTED a triangle has an area of 72 unites squared the base is 4 units tall how tall is the height Based on the conversation between JackThayer and Milton the reader can infer-- Given the right angle triangle shown here, calculate:1. a) sin(a)172. a) sin(b)8b) cos(a)bb) cos(b)15c) tan(a)c) tan(b)Write trig. ratios for each. T. palms has a basis of $20,000 for her one-third interest in a partnership. the partnership has no liabilities, cash of $22,000, and property with a partnership basis of $38,000 and fair market value of $44,000. for her one-third interest, palms receives the cash of $22,000. if the partnership elects sec. 754 optional basis adjustment, what is the basis of the retained partnership property? 4x + 11y = -3 and -6x = 18y - 6 (substitution method) Choose the sentence that is grammatically correct. Est-il du papier toilette dans la salle de bain. Est-ce qu'il y a du papier toilette dans la salle de bain? Est-ce qu'y a-t-il du papier toilette dans la salle de bain? Est-ce qu'il a y du papier toilette dans la salle de bain? Find the vertex and symmetry for each problem.y=3X^2+6X+1y=2x^2-8x+3y=x^2-10x+25y=5(x+2)^2+1y= -2x^2+8x+7y=23x^2-7y= -(x+1)^2y=3x^2-24x+6y=(x-5)^2+3y=2.5x^2-15x+9 Plz help me well mark brainliest if correct....?.?.?.? i need help with geometry Can someone please answer these 4 questions they dont look hard- but I just need to understand please!! (If you can provide the answer please!!!!!)